Behavior of the logarithm function in the complex domain ....

In summary, the conversation discusses the usefulness of the integral $\displaystyle\int_{0}^{\infty}\frac{\ln x}{x^{2}+a^{2}}\ dx$ in complex analysis and the different definitions of the logarithm in the complex field. Two alternatives, a) and b), are explored and the conclusion is made that the principle of argument is necessary to answer the posed questions.
  • #1
chisigma
Gold Member
MHB
1,628
0
In...... it has beenshown in an elementary way that is ...

$\displaystyle\int_{0}^{\infty}\frac{\ln x}{x^{2}+a^{2}}\ dx = \frac{\pi\ \ln a}{2\ a}\ (1)$The integral (1) isvery useful to shed light on the behavior of the logarithm functionin the complex field. Let's suppose we want to solve the integralusing the methods of complex analysis, which means to integrate thefunction $\displaystyle f(z) = \frac{\ln z}{z^{2} + a^{2}}$ along theclosed path shown in figure...

View attachment 2794
... looking for the limit when the great circle tends to infinity and the small circle tends to zero. According to the residue theorem is...$\displaystyle\int_{C} \frac{\ln z}{z^{2} + a^{2}}\ d z = 2\ \pi\ i\ \sum_{k}r_{k}\ (2)$... where r are the residues of the poles of f (z) that fall inside the path. The polesof f (z) are z = a i and z = - a i , so that the problem lies in theexact definition of the logarithm in the complex field. According tomost of the 'sacred texts' the logarithm of a complex number z is given by $\displaystyle \ln z = \ln |z| + i\ \theta$ , where in anycase is $- \pi < \theta \le \pi$. We call this alternative as'a)'. According to the alternative a) it will be ...$\displaystyle r_{1}= \lim_{z \rightarrow a\ i} \frac{\ln z}{z + a\ i} = \frac{i\ \frac{\pi}{2}}{2\ a\ i} = \frac{\pi}{4\ a}$$\displaystyle r_{2}= \lim_{z \rightarrow - a\ i} \frac{\ln z}{z - a\ i} = \frac{- i\ \frac{\pi}{2}}{- 2\ a\ i} =\frac{\pi}{4\ a}\ (3)$... so that is...$\displaystyle\int_{C} \frac{\ln z}{z^{2} + a^{2}}\ d z = 2\ \pi\ i\ (r_{1} +r_{2}) = i\ \frac{\pi^{2}}{a}\ (4)$

On the other hand, always in agreement with the alternative a) is ...

$\displaystyle \int_{C} \frac{\ln z}{z^{2} + a^{2}}\ d z = \int_{r}^{R} \frac{\ln x}{x^{2} + a^{2}} + i\ R\ \{ \int_{0}^{\pi} \frac{e^{i\ \theta}\ (\ln R + i\ \theta)}{a^{2} + R^{2}\ e^{2\ i\ \theta}}\ d \theta + \int_{- \pi}^{0} \frac{e^{i\ \theta}\ (\ln R + i\ \theta)}{a^{2} + R^{2}\ e^{2\ i\ \theta}}\ d \theta\ \} - \int_{r}^{R} \frac{\ln x}{x^{2} + a^{2}} - i\ r\ \{ \int_{0}^{\pi} \frac{e^{i\ \theta}\ (\ln r + i\ \theta)}{a^{2} + r^{2}\ e^{2\ i\ \theta}}\ d \theta + \int_{- \pi}^{0} \frac{e^{i\ \theta}\ (\ln R + i\ \theta)}{a^{2} + R^{2}\ e^{2\ i\ \theta}}\ d \theta\ \}\ (5)$

Now it is not difficult to see that for r tending to zero and R tends to infinity the integral (5) tends to zero, so that comparing (4) and (5) we arrive at the 'identity' $\displaystyle i\ \frac{\pi^{2}}{a}=0$! (Tmi) ...

If this is so is evident that either the theorem of residues, or in the alternative a) is wrong. Since giving up the residue theorem would be painful, let's consider a different alternative: the logarithm of a complex number z is given by $\displaystyle \ln z = \ln |z| + i\ \theta$ , where is $0 < \theta \le 2\ \pi$. We call this alternative as 'b)'. According to the alternative b) it will be ...

$\displaystyle r_{1}= \lim_{z \rightarrow a\ i} \frac{\ln z}{z + a\ i} = \frac{i\ \frac{\pi}{2}}{2\ a\ i} = \frac{\pi}{4\ a}$$\displaystyle r_{2}= \lim_{z \rightarrow - a\ i} \frac{\ln z}{z - a\ i} = \frac{i\ \frac{3\ \pi}{2}}{- 2\ a\ i} = - \frac{3\ \pi}{4\ a}\ (6)$

... so that is...$\displaystyle\int_{C} \frac{\ln z}{z^{2} + a^{2}}\ d z = 2\ \pi\ i\ (r_{1} +r_{2}) = - i\ \frac{\pi^{2}}{a}\ (7)$

On the other hand, always in agreement with the alternative a) is ...

$\displaystyle \int_{C} \frac{\ln z}{z^{2} + a^{2}}\ d z = \int_{r}^{R} \frac{\ln x}{x^{2} + a^{2}} + i\ R\ \int_{0}^{2\ \pi} \frac{e^{i\ \theta}\ (\ln R + i\ \theta)}{a^{2} + R^{2}\ e^{2\ i\ \theta}}\ d \theta - \int_{r}^{R} \frac{\ln x}{x^{2} + a^{2}} - 2\ \pi \ i\ \int_{r}^{R} \frac{d x}{a^{2} + x^{2}}\ dx - i\ r\ \int_{0}^{\pi} \frac{e^{i\ \theta}\ (\ln r + i\ \theta)}{a^{2} + r^{2}\ e^{2\ i\ \theta}}\ d \theta\ (8)$

Now it is not difficult to see that for r tending to zero and R tends to infinity the integral (8) tends to $\displaystyle - i\ \frac{\pi^{2}}{a}$ so that comparing (7) and (8) we arrive at $\displaystyle - i\ \frac{\pi^{2}}{a}=- i\ \frac{\pi^{2}}{a}$, something that fills us with happiness! (Happy)...

Add consideration on which of the alternatives a) or b) is 'preferable' is clearly superfluous ...

Since this is a note, comments and clarifications have to be made on the commentary ...

Kind regards

$\chi$ $\sigma$
 

Attachments

  • MHB03.PNG
    MHB03.PNG
    6.1 KB · Views: 88
Last edited by a moderator:
Physics news on Phys.org
  • #2
It is significant that this note has been taken in response to the post...

http://mathhelpboards.com/analysis-50/complex-logarithm-11275.html#post52548

...and that new fuel has been provided by the post ...

http://mathhelpboards.com/analysis-50/contour-integral-method-query-11457.html#post53618...produced by the same user ... congratulations to Fermat! (Yes) ...In the previous post we examined two different alternatives to thedefinition of the logarithm in the complex field: alternative a),accepted by most of the literature, and alternative b) minority in the literature but strongly supported by the writer. The strict answer to the questions posed by Fermat requires the use of powerful complex analysis theorem known as the principle of argument, which states that, given an f (z) meromorphic in a region included in a closed path $\gamma$, is...

$\displaystyle \int_{\gamma} \frac{f^{\ '}(z)}{f(z)}\ d z = 2\ \pi\ i\ (n - m)\ (1)$

... where n is the number of zeros and m the number of poles of f (z) inside $\gamma$, each of which taken in agreement with its multiplicity.The proof of the theorem is quite easy if you start with the essential requirements ...

a) f (z) must be in the form $\displaystyle f(z)= \frac{p(z)}{q(z)}$ , where p (z) is a polynomial of degree n and q (z) be a polynomial of degree m ...

b) function ln [f (z)] is analytic everywhere except the zeros and poles of f (z) and its derivative is $\displaystyle \frac{d}{d z} \ln [f(z)] = \frac{f^{\ '} (z)}{f(z)}$ ...

c) given two complex numbers a and b, both different from 0, apply the base relations $\displaystyle \ln (a\ b) = \ln a + \ln b$ and $\displaystyle \ln \frac{a}{b}= \ln a - \ln b$...

It should be noted that the property c) is compatible with the alternative b) and not with the alternative a). Given the hypotheses a), b) and c), the proof proceeds without any problems. First is ...

$\displaystyle f(z) = \frac{a_{0}\ (z - a_{1})\ (z-a_{2})...(z-a_{n})}{b_{0}\ (z - b_{1})\ (z - b_{2})...(z-b_{m})}\ (2)$

From (2) and the properties b) and c) we obtain...

$\displaystyle \frac{d}{d z} [\ln f(z)] = \frac{f^{\ '}(z)}{f(z)} = \sum_{k=1}^{n} \frac{1}{z-a_{k}} - \sum_{k=1}^{m} \frac{1}{z-b_{k}}\ (3)$

... and by (3), using the residue theorem, we arrive at (1). Very interesting is the simplest case of all: f(z) = z and $\gamma$ is the unit circle. In that case we arrive to the result...

$\displaystyle \int_{|z=1|} \frac{d z}{z} = 2\ \pi\ i\ (4)$

... the correctness of which we all agree ...

Last not useless observation to make is that it is forced into a crucial choice: if the alternative a) is wrong, then the alternative b) is good and good is also the principle of the argument ... if the alternative a) is good then the alternative b) is wrong and wrong is also the principle of the argument. Now is the chance to point out that the principle of argument in recent decades has made possible to arrive at valuable results in the field of complex analysis, so (Thinking)...

Kind regards

$\chi$ $\sigma$
 
Last edited:

Related to Behavior of the logarithm function in the complex domain ....

What is the logarithm function in the complex domain?

The logarithm function in the complex domain is a mathematical function that maps a complex number to another complex number. It is defined as the inverse of the exponential function, and is denoted as log(z).

What is the behavior of the logarithm function in the complex domain?

The behavior of the logarithm function in the complex domain is quite different from its behavior in the real domain. In the real domain, the logarithm function is a single-valued function, but in the complex domain, it becomes a multi-valued function due to the presence of an infinite number of complex roots.

What is the principal branch of the logarithm function in the complex domain?

The principal branch of the logarithm function in the complex domain is the branch that is chosen to be the "main" branch. It is usually defined as the branch where the imaginary part of the logarithm is between -π and π. This is the most commonly used branch of the logarithm function in the complex domain.

How does the complex logarithm behave near its branch points?

The complex logarithm function has branch points at z=0 and z=∞. Near these branch points, the function exhibits a behavior known as "branch cut". This means that the function is not continuous at these points, and the values of the function on either side of the branch cut differ by a constant multiple of 2πi.

What are the applications of the complex logarithm function?

The complex logarithm function has many applications in mathematics and physics. It is used in solving complex equations, calculating the roots of complex numbers, and in the study of complex functions and their behavior. It is also used in signal processing, control systems, and in many other areas of science and engineering.

Similar threads

  • Calculus and Beyond Homework Help
Replies
3
Views
614
  • Advanced Physics Homework Help
Replies
19
Views
957
Replies
4
Views
499
Replies
4
Views
810
  • Calculus and Beyond Homework Help
Replies
9
Views
276
  • Introductory Physics Homework Help
Replies
4
Views
806
  • Calculus
Replies
29
Views
874
  • Introductory Physics Homework Help
Replies
6
Views
1K
  • Introductory Physics Homework Help
Replies
17
Views
452
  • Introductory Physics Homework Help
Replies
5
Views
381
Back
Top